Đến nội dung

Hình ảnh

TOPIC [MỘT SỐ BÀI TOÁN SỐ HỌC]

- - - - -

  • Please log in to reply
Chủ đề này có 8 trả lời

#1
KietLW9

KietLW9

    Đại úy

  • Điều hành viên THCS
  • 1737 Bài viết

$\textbf{Bài toán 1 (Brazil National Olympiad 2009).}$ Cho $p,q$ là các số nguyên tố thỏa mãn $q=2p+1$. Chứng minh rằng tồn tại một bội của $q$ mà tổng các chữ số trong biểu diễn thập phân của nó không vượt quá $3$.

$\textbf{Lời giải.}$

Rõ ràng $q\geq 5$. Xét $q=5$ thì $10$ là bội của $5$ và tổng của các chữ số của $10$ là $1$ (thỏa mãn)

Nếu $q>5$ thì $(q,10)=1$. Áp dụng định lý Fermat nhỏ, ta được: $10^{q-1}\equiv 1 (\text{mod q})\Rightarrow 10^{2p}\equiv 1 (\text{mod q})\Rightarrow q | (10^p+1)(10^p-1)$

$\textbf{Trường hợp 1.}$ $q|10^p+1$ thì rõ ràng $10^p+1$ là một bội của $q$ mà tổng các chữ số trong biểu diễn thập phân của nó là $2$ nên bài toán được chứng minh

$\textbf{Trường hợp 2.}$ $q|10^p-1$. Gọi $h=ord_q(10)$ thì $h=1$ hoặc $h=p$. Rõ ràng $h=1$ vô lí vì nếu $h=1$ thì $q|9$. Do đó $h=p$ hay nói cách khác $p$ là cấp của $10$ theo modulo $q$

Suy ra khi ta đem chia các số $10^1,10^2,...,10^p$ cho $q$ thì chúng sẽ lần lược nhận các số dư phân biệt theo thứ tự là $r_1,r_2,...,r_p$ với $1\leq r_i\leq 2p(i=\overline{1,p})$

Nếu $r_i=p$ thì $2.10^i+1\equiv 2r_i+1=2p+1\equiv 0(\text{mod q})$ nên đây cũng là một bội của $q$ có tổng các chữ số là $3$. Tương tự với $r_i=2p$ thì ta cũng thấy thỏa mãn

Trong trường hợp trong $p$ số trên không có số nào khi chia $q$ có số dư là $p$ hoặc $2p$ thì ta phân hoạch $2p-2$ số dư còn lại trong tập hợp số dư thành $p-1$ tập con như sau $$\left \{ 1,2p-1 \right \},\left \{ 2,2p-2 \right \},...,\left \{ p-1,p+1 \right \}$$

Có $p$ số mà chỉ có $p-1$ tập con nên theo nguyên lí Dirichlet thì tồn tại hai số $r_i,r_j$ sao cho $r_i+r_j=2p$. Tức là $10^i+10^j+1 \equiv 2p+1\equiv 0(\text{mod q})$. Mà $10^i+10^j+1$ là một số biểu diễn trong hệ thập phân có tổng các chữ số là $3$ nên trường hợp nãy vẫn thỏa mãn

Vậy ta có điều phải chứng minh


Bài viết đã được chỉnh sửa nội dung bởi KietLW9: 14-08-2022 - 07:14

Trong cuộc sống không có gì là đẳng thức , tất cả đều là bất đẳng thức  :ukliam2:   :ukliam2: 

 

 

$\text{LOVE}(\text{KT}) S_a (b - c)^2 + S_b (c - a)^2 + S_c (a - b)^2 \geqslant 0\forall S_a,S_b,S_c\geqslant 0$

 

 

 


#2
narutosasukevjppro

narutosasukevjppro

    Trung sĩ

  • Thành viên
  • 131 Bài viết

a góp mấy bài được k e  :like  :D



#3
KietLW9

KietLW9

    Đại úy

  • Điều hành viên THCS
  • 1737 Bài viết

Được anh ạ


Trong cuộc sống không có gì là đẳng thức , tất cả đều là bất đẳng thức  :ukliam2:   :ukliam2: 

 

 

$\text{LOVE}(\text{KT}) S_a (b - c)^2 + S_b (c - a)^2 + S_c (a - b)^2 \geqslant 0\forall S_a,S_b,S_c\geqslant 0$

 

 

 


#4
KietLW9

KietLW9

    Đại úy

  • Điều hành viên THCS
  • 1737 Bài viết

$\textbf{Bài toán 2 (THTT T10/516).}$ Tìm tất cả các cặp số nguyên tố $p,q$ để $\frac{3^q+1}{p^2+1}$ và $\frac{3^p+1}{q^2+1}$ đồng thời là các số nguyên dương

 

$\textbf{Lời giải.}$ Không mất tính tổng quát, ta có thể giả sử $p\leq q$

 

$\textbf{Trường hợp 1.}$ $p=2$ khi đó $5|3^q+1$ và $q^2+1|10$ nên dễ dàng tìm được $q=2$

 

$\textbf{Trường hợp 2.}$ $p$ là số nguyên tố lẻ thì rõ ràng $p^2+1$ sẽ chứa thừa số nguyên tố lẻ. Gọi $r$ là một ước nguyên tố lẻ của $p^2+1$ thì $3^q+1\equiv 0(\text{mod r})\Rightarrow (-3)^{q}\equiv 1(\text{mod r})$

Gọi $h=ord_r(-3)$ thì $h|q$. Nếu $h=1$ thì ta suy ra vô lí ngay nên $h=q$ (Lưu ý rằng $p^2+1$ không chứa thừa số nguyên tố $3$ nên $(r,3)=1$)

Mà theo định lý Fermat nhỏ ta cũng có được $(-3)^{r-1}\equiv 1(\text{mod r})$ nên $q|r-1$. 

Như vậy mọi ước nguyên tố lẻ của $p^2+1$ đều đồng dư với $1$ theo modulo $q$

Mặt khác $p^2+1=2m$ ($m$ là số nguyên dương lẻ) nên $p^2+1\equiv 2(\text{mod q})\Rightarrow q|(p+1)(p-1)$

Do $p-1<p\leq q$ nên $q|p+1$. Lúc này ta có $p\leq q\leq p+1$ nên $q=p+1$ suy ra q là số nguyên tố chẵn (Vô lí)

Vậy $(p,q)=(2,2)$

$\textbf{Bài toán tương tự.}$ Tìm tất cả các cặp số nguyên tố $p,q$ để $\frac{(2p^2-1)^q+1}{p+q}$ và $\frac{(2q^2-1)^p+1}{p+q}$ đồng thời là các số nguyên dương


Bài viết đã được chỉnh sửa nội dung bởi KietLW9: 16-08-2022 - 14:37

Trong cuộc sống không có gì là đẳng thức , tất cả đều là bất đẳng thức  :ukliam2:   :ukliam2: 

 

 

$\text{LOVE}(\text{KT}) S_a (b - c)^2 + S_b (c - a)^2 + S_c (a - b)^2 \geqslant 0\forall S_a,S_b,S_c\geqslant 0$

 

 

 


#5
KietLW9

KietLW9

    Đại úy

  • Điều hành viên THCS
  • 1737 Bài viết

$\textbf{Bài toán 3 (THTT T10/517).}$ Tìm các số nguyên dương $k$ và số nguyên tố $p$ thỏa mãn $$k!=(p^3-1)(p^3-p)(p^3-p^2)$$

$\textbf{Lời giải.}$ Xét $p=2$ thì $k!=168$. Trường hợp này không có số $k$ nào thỏa mãn.

Xét $p>2$ thì $k!=p^3(p-1)^3(p+1)(p^2+p+1)$ nên $v_p(k!)=3$

Mặt khác theo công thức Lengendre thì $v_p(k!)=\sum_{i=1}^{n}\left [ \frac{k}{p^i} \right ]$ với $n$ là số thỏa mãn $p^n\leq k< p^{n+1}$

Nếu $n\geq 2$ thì $k\geq p^2\geq 3p$. Nếu $n=1$ thì $3=\left [ \frac{k}{p} \right ]\Rightarrow k\geq 3p$. Vì thế ta luôn có $k\geq 3p>2(p+1)\Rightarrow (p+1)^2|k!$

$\Rightarrow p+1|p^3(p-1)^3(p^2+p+1)$

Mà $(p+1,p^2+p+1)=1$ nên $p^3(p-1)^3\equiv 0(\text{mod p+1})$

Mà $p^3(p-1)^3\equiv (-1)^3.(-2)^3\equiv 8(\text{mod p+1})\Rightarrow p+1|8$ nên $p=7$ hoặc $p=3$

Hai số này đều không thỏa nên ta kết luận không tồn tại số nguyên dương $k$ và số nguyên tố $p$ thỏa mãn đề bài


Trong cuộc sống không có gì là đẳng thức , tất cả đều là bất đẳng thức  :ukliam2:   :ukliam2: 

 

 

$\text{LOVE}(\text{KT}) S_a (b - c)^2 + S_b (c - a)^2 + S_c (a - b)^2 \geqslant 0\forall S_a,S_b,S_c\geqslant 0$

 

 

 


#6
KietLW9

KietLW9

    Đại úy

  • Điều hành viên THCS
  • 1737 Bài viết

$\textbf{Bài toán 4.}$ Tìm tất cả các cặp số nguyên dương $(a,b)$ sao cho $a,b>1$ và $b^a|a^b-1$

 

$\textbf{Lời giải.}$

$\textbf{Bổ đề.}$ Cho $a,b$ là các số nguyên, $n$ là số nguyên dương và $p$ là số nguyên tố thỏa mãn $(n,p-1)=1$. Khi đó nếu $p|a^n-b^n$ thì $ p|a-b$

Gọi $p$ là ước nguyên tố nhỉ nhất của $b$

Áp dụng bổ đề cho bài toán trên, ta được: $p|a-1$ do $p$ là ước nguyên tố nhỏ nhất của $b$ nên $(b,p-1)=1$

Nếu $p$ lẻ thì áp dụng bổ đề LTE, ta được: $v_p(a^b-1)=v_p(a-1)+v_p(b)\geqslant v_p(b^a)=av_p(b)\Rightarrow v_p(b)(a-1)\leq v_p(a-1)$

Mà $v_p(b)(a-1)\geq a-1$ nên $a-1\leq v_p(a-1)$. Đây là một điều hết sức vô lí nên $p=2$

Lúc đó $b$ chẵn và $a$ lẻ

Tiếp tục sử dụng bổ đề LTE cho $p=2$, ta được: $av_2(b)\leq v_2(a^b-1)=v_2(a+1)+v_2(a-1)+v_2(b)-1\Rightarrow a\leq v_2(b)(a-1)+1=v_2(a+1)+v_2(a-1)$

Điều này chỉ xảy ra khi $a=3$. Lúc đó $v_2(b)=1$. Đặt $b=2k$ với $k$ là số nguyên dương lẻ thì ta cần tìm số $k$ sao cho $(2k)^3|3^{2k}-1$

Nếu $k>1$ thì rõ ràng $k$ tồn tại ước nguyên tố lẻ nên gọi $q$ là ước nguyên tố lẻ nhỏ nhất của $k$. Khi đó theo bổ đề chứng minh ở trên thì $q|3^{2k}-1\Rightarrow q|9-1=8$. Vô lí do $q$ lẻ

Do đó $k=1$ suy ra $b=2$

Vậy $a=3,b=2$


Trong cuộc sống không có gì là đẳng thức , tất cả đều là bất đẳng thức  :ukliam2:   :ukliam2: 

 

 

$\text{LOVE}(\text{KT}) S_a (b - c)^2 + S_b (c - a)^2 + S_c (a - b)^2 \geqslant 0\forall S_a,S_b,S_c\geqslant 0$

 

 

 


#7
KietLW9

KietLW9

    Đại úy

  • Điều hành viên THCS
  • 1737 Bài viết

$\textbf{Bài toán 5.}$ Cho $n$ là số nguyên dương lớn hơn $1$ sao cho $n|6^n+7^n$. Chứng minh rằng $13|n$

$\textbf{Lời giải.}$ Rõ ràng $(n,42)=1$. Gọi $p$ là ước nguyên tố nhỏ nhất của $n$

Vì $(p,7)=1$ nên tồn tại số nguyên $p'$ sao cho $7p'\equiv 1(\text{mod p})$

Ta có: $6^n\equiv -7^n(\text{mod p})\Rightarrow (6p')^n\equiv -(7p')^n\equiv -1(\text{mod p})\Rightarrow (6p')^{2n}\equiv 1(\text{mod p})$

Gọi $h=\text{ord}_p(6p')$ thì $h|p-1$ và $h|2n$

Nếu $h$ lẻ thì $h<p$ và $h|n$ nên $h=1$

Nếu $h$ chẵn thì $h=2l$ suy ra $l<p$ và $l|n$ do đó $l=1$ và $h=2$

Vậy ta đều có $(6p')^2\equiv 1(\text{mod p})\Rightarrow (6^2-7^2)p'^2\equiv 0(\text{mod p})$

Mà rõ ràng $(p',p)=1$ do đó $p=13$

Vậy $13|n$


Trong cuộc sống không có gì là đẳng thức , tất cả đều là bất đẳng thức  :ukliam2:   :ukliam2: 

 

 

$\text{LOVE}(\text{KT}) S_a (b - c)^2 + S_b (c - a)^2 + S_c (a - b)^2 \geqslant 0\forall S_a,S_b,S_c\geqslant 0$

 

 

 


#8
KietLW9

KietLW9

    Đại úy

  • Điều hành viên THCS
  • 1737 Bài viết

$\textbf{Bài toán 6.}$ Xét $p$ là số nguyên tố có dạng $4k+1$ trong đó $k$ là số nguyên dương thỏa mãn $p^2|2^{p-1}-1$. Gọi $q$ là ước nguyên tố lớn nhất của $2^p-1$ Chứng minh rằng: $2^q > (6p)^p$

 

$\textbf{Lời giải.}$ 

Ta có phân tích tiêu chuẩn của $2^p-1$ như sau: $2^p-1=q_{1}^{a_1}q_{2}^{a_2}...q_{k}^{a_k}$ trong đó $q_1,q_2,...,q_k$ là các ước nguyên tố lẻ phân biệt của $2^p-1$.

Đầu tiên ta sẽ chứng minh $q_i\equiv 1(\text{mod p}),\forall i=\overline{1,k}$

Thật vậy, ta có: $q_i|2^{q_i-1}-1$ nên $\left\{\begin{matrix}ord_{p_i}(2)|p & \\ ord_{p_i}(2)|q_i-1 & \end{matrix}\right.$ 

Từ đây dễ dàng suy ra $ord_{p_i}(2)=p$ nên ta có điều phải chứng minh

Đặt $q_i=m_ip+1$ trong đó $m_i \in \mathbb{Z}^+,\forall i =\overline{1,k}$

Kết hợp với $p^2|2^p-2$ ta suy ra $\prod_{i=1}^{k}(m_ip+1)^{a_i}\equiv 1$ (mod $p^2$). Mà $(m_ip+1)^{a_i}\equiv a_im_ip+1$ (mod $p^2$) nên $\prod_{i=1}^{k}(1+a_1m_ip)\equiv 1$ (mod $p^2$) nên $ \sum_{i=1}^{k}a_im_i\equiv 0$ (mod $p$)

Mà lại có $q_i|2^p-1\Rightarrow (\frac{2}{q_i})=1$ nên $q_i\equiv 1,7(\text{mod 8})$ do đó $8|m_i$ hoặc $6|m_ip$ nên $m_i\geq 6$

Xét $q$ là ước nguyên tố lớn nhất của $2^p-1$ sao cho $q=mp+1$ thì rõ ràng $m$ là chỉ số lớn nhất trong các số $m_i$ khi đó $2^q>2^{pm}>(2^p-1)^m>(6p)^{(a_1+a_2+...+a_k)m}>(6p)^{\sum_{i=1}^{k}a_im_i}\geq (6p)^p$

Vậy ta có điều phải chứng minh


Trong cuộc sống không có gì là đẳng thức , tất cả đều là bất đẳng thức  :ukliam2:   :ukliam2: 

 

 

$\text{LOVE}(\text{KT}) S_a (b - c)^2 + S_b (c - a)^2 + S_c (a - b)^2 \geqslant 0\forall S_a,S_b,S_c\geqslant 0$

 

 

 


#9
KietLW9

KietLW9

    Đại úy

  • Điều hành viên THCS
  • 1737 Bài viết

$\textbf{Bài toán 7.}$ Chứng minh rằng với mọi $n$ nguyên dương luôn tồn tại $m$ để $$7^n|3^m+5^m-1$$

$\textbf{Lời giải.}$ Chọn $m=7^{n-1}$ và sử dụng LTE, ta có: $$v_7(8^m-1)=v_7(8-1)+v_7(m)=n\Rightarrow 7^n|8^m-1=(2^m-1)(4^m+2^m+1)$$

Mặt khác $m$ không là bội của $3$ nên $7^n \not| 2^m-1\Rightarrow 7^n|4^m+2^m+1$

Kết hợp với $m$ lẻ ta có: $v_7(4^m+3^m)=v_7(3+4)+v_7(m)=n\Rightarrow 4^m\equiv -3^m$ (mod $7^n$)

Tương tự ta cũng có: $2^m\equiv -5^m$ (mod $7^n$) nên $3^m+5^m-1\equiv 0$ (mod $7^n$)

$\textbf{Bài toán 8.}$ Cho $p$ là một số nguyên tố lẻ. Chứng minh rằng mọi ước nguyên tố của $\frac{p^{2p}+1}{p^2+1}$ đều có dạng $4k+1$

$\textbf{Lời giải.}$ Rõ ràng với $p$ lẻ thì $\frac{p^{2p}+1}{p^2+1}$ lẻ nên tất cả các ước nguyên tố của $\frac{p^{2p}+1}{p^2+1}$ đều lẻ. Gọi $q$ là một trong số chúng $\Rightarrow q|p^{4p}-1$

Gọi $h$ là cấp của $p$ theo modulo $q$ thì $h|4p$ nên $h \in \left \{ 1;2;4;p;2p;4p \right \}$

Nếu $h \in \left \{ 1;2;p;2p \right \}$ thì ta luôn có $p^{2p}\equiv 1(\text{mod q})$, vô lí với $p^{2p}\equiv -1(\text{mod q})$

* Nếu $h=4p$ thì $q\equiv 1(\text{mod 4p})$

* Nếu $h=4$ thì $q$ vẫn có dạng $4k+1$

Do đó ta có điều phải chứng minh

$\textbf{Bài toán 9.}$ Cho $a,m,n$ là các số nguyên lớn hơn $1$. Chứng minh rằng nếu $a^2+a+1|(a+1)^m+a^n$ thì $3|m+n$


Trong cuộc sống không có gì là đẳng thức , tất cả đều là bất đẳng thức  :ukliam2:   :ukliam2: 

 

 

$\text{LOVE}(\text{KT}) S_a (b - c)^2 + S_b (c - a)^2 + S_c (a - b)^2 \geqslant 0\forall S_a,S_b,S_c\geqslant 0$

 

 

 





1 người đang xem chủ đề

0 thành viên, 1 khách, 0 thành viên ẩn danh